LSAT and Law School Admissions Forum

Get expert LSAT preparation and law school admissions advice from PowerScore Test Preparation.

User avatar
 Dave Killoran
PowerScore Staff
  • PowerScore Staff
  • Posts: 5850
  • Joined: Mar 25, 2011
|
#43483
Complete Question Explanation
(The complete setup for this game can be found here: lsat/viewtopic.php?t=7431)

The correct answer choice is (A)

If G is not assigned to a team, then each team features one experienced plumber and one inexperienced plumber. From the fourth rule, T must be assigned to K. F is the next logical plumber to examine due to the number of restrictions on F, and of the remaining inexperienced plumbers F must be assigned to a team with S. Because J cannot be assigned to a team with R, J must be assigned to a team with V, leaving M to team with R:
J95_Game_#4_#23_diagram 1.png
Answer choice (A) is thus correct.

Get the most out of your LSAT Prep Plus subscription.

Analyze and track your performance with our Testing and Analytics Package.